Optimization: square inscribed in a square

Click For Summary
To prove that the inscribed square with minimum area has edges of length (1/2)L√2, one can start by labeling the vertices of the outer square and defining the vertices of the inscribed square with a variable x. The area of the inscribed square can be expressed in terms of x, and finding its minimum involves calculating the derivative of the area function. It is clarified that a square cannot be rotated 360 degrees while remaining inscribed with its corners touching the sides of the outer square. The optimal inscribed square is formed by connecting the midpoints of the sides of the larger square.
enian
Messages
23
Reaction score
0

Homework Statement


Each edge of a square has length L. Prove that among all squares inscribed in the given square, the one of minimum area has edges of length \frac{1}{2}L\sqrt{2}


Homework Equations





The Attempt at a Solution


I started by drawing a square of sides L. Then labeled the vertices: (0,0) (L,0) (0,L) (L,L) then drew an inscribed square with variable x and the vertices were: (x,0) (L,x) (L-x,L) (0,L-x)
Then from this I set the distance of all the lines between each of these inscribed triangles vertices equal to one anotehr, to determine what values of X would work for the equation.
I determined that x=x. Is this true? Could you rotate a square 360 degrees while still being inscribed within a square?

I was stumped at this part, but if it's true. Then area of the inscribed square is going to be the distance of one of the inscribed squares length squared.
such as.. ((x-L)^{2}+(x)^{2})^{2}
Then i would take a derivative of this and determine the minimum point?
Is this right?
 
Physics news on Phys.org
"Could you rotate a square 360 degrees while still being inscribed within a square?"

No, you cannot, if the four corners are still touching the sides. You must have made some small mistake.

Just find the area of the inscribed square as the sum of two triangles, by considering the vertices in proper order. Or you can find the dist between the parallel pairs of lines to get the area in terms of x. This one'd be easier. Whatever you do, you should be able to show the given result.

(The result shows that the least square is obtained by joining the mid-points of the bigger square.)
 
Question: A clock's minute hand has length 4 and its hour hand has length 3. What is the distance between the tips at the moment when it is increasing most rapidly?(Putnam Exam Question) Answer: Making assumption that both the hands moves at constant angular velocities, the answer is ## \sqrt{7} .## But don't you think this assumption is somewhat doubtful and wrong?

Similar threads

  • · Replies 1 ·
Replies
1
Views
1K
Replies
3
Views
2K
  • · Replies 9 ·
Replies
9
Views
2K
  • · Replies 2 ·
Replies
2
Views
2K
  • · Replies 4 ·
Replies
4
Views
2K
  • · Replies 2 ·
Replies
2
Views
1K
  • · Replies 1 ·
Replies
1
Views
2K
  • · Replies 1 ·
Replies
1
Views
2K
Replies
12
Views
2K
Replies
4
Views
2K